Sieht jemand, der in ein Schwarzes Loch fällt, das Ende des Universums?

Diese Frage wurde von Kann Materie wirklich durch einen Ereignishorizont fallen? . Wenn Sie die Schwarzschild-Koordinatenzeit für irgendetwas, Materie oder Licht, berechnen, um den Ereignishorizont zu erreichen, ist das Ergebnis bekanntermaßen unendlich. Dies impliziert, dass das Universum um eine unendliche Zeit altert, bevor jemand, der in das Schwarze Loch fällt, den Ereignishorizont erreicht. Könnte diese Person also sehen, wie das Universum um eine unendliche Zeit altert?

Um genauer zu sein, nehmen Sie an, der Beobachter beginnt zu einem bestimmten Zeitpunkt aus der Ruhe zu fallen t = 0 und etwas Anfangsabstand r > r s . Wenn wir einige Zeit warten T Dann strahlen Sie einen Lichtstrahl auf den fallenden Beobachter. Wird der Lichtstrahl den fallenden Beobachter immer erreichen, bevor er den Ereignishorizont überschreitet? Wenn nicht, was ist die Formel für die längste Zeit T dass wir warten und trotzdem sicher sein können, dass der Strahl den Betrachter trifft? Wenn T nicht begrenzt ist, bedeutet dies, dass der Beobachter tatsächlich das Ende des Universums sehen könnte.

Mir fällt ein qualitatives Argument für eine Obergrenze ein T , aber ich bin mir nicht sicher, wie stichhaltig mein Argument ist. Die richtige Zeit, in der der Beobachter zum Ereignishorizont fällt, ist endlich - nennen Sie dies τ . Die richtige Zeit für den Lichtstrahl, den Horizont zu erreichen, ist Null, daher erreicht der Lichtstrahl den Beobachter nur dann, bevor er den Ereignishorizont überquert T < τ . Somit T ist begrenzt und der Beobachter sieht das Ende des Universums nicht.

Ich denke, ein strengerer Ansatz wäre, die Bewegungsgleichungen (in den Schwarzschild-Koordinaten) für den fallenden Beobachter und den Lichtstrahl zu bestimmen und dann die Bedingung zu finden, damit das Licht den fallenden Beobachter in einiger Entfernung erreicht ϵ aus dem Ereignishorizont. Dann nehmen Sie die Grenze als ϵ 0 . Im Prinzip scheint das einfach, aber in der Praxis hat mich die Algebra schnell besiegt. Selbst für einen Lichtstrahl ist die radiale Entfernung:Zeit-Gleichung keine geschlossene Form (Wolfram behauptet, dass es die benötigt W Funktion) und für den fallenden Beobachter ist die Rechnung noch schwieriger.

Aus diesem Diagramm in der ausgewählten Antwort ist ersichtlich, dass sich sowohl der weit entfernte Beobachter als auch der einfallende Beobachter zu einem beliebigen Zeitpunkt in der Zukunft immer noch außerhalb des Schwarzen Lochs befinden (es gibt eine zeitgleiche gestrichelte Linie, die die beiden verbindet). Da das Schwarze Loch in endlicher Zeit verdunstet, bedeutet dies, dass der einfallende Beobachter niemals den Ereignishorizont überschreiten wird. Aber ja, der einfallende Beobachter in endlicher Eigenzeit überquert entweder den Horizont (wenn es keine Hawking-Strahlung gibt) oder sieht das BH verschwinden. Er wird keine unendliche Zukunft sehen (aber wird die unendliche Zukunft beeinflussen).

Antworten (9)

Ich würde empfehlen, sich bei solchen Fragen von Schwarzschild-Koordinaten fernzuhalten. Alle klassischen Unendlichkeiten (abgesehen vom Firewall-Paradoxon), die mit dem Ereignishorizont zu tun haben, sind auf schlechte Koordinatenwahl zurückzuführen. Sie möchten ein Koordinatensystem verwenden, das am Horizont regelmäßig ist, wie Kruskal-Szekeres . Schauen Sie sich in der Tat das Kruskal-Szekeres-Diagramm an:

Kruskal-Szekeres-Diagramm(Quelle: Wikipedia)

Dies ist die maximal erweiterte Schwarschild-Geometrie, kein physisches Schwarzes Loch, das sich aus einem Sternkollaps bildet, aber die Unterschiede sollten uns für diese Frage nicht stören. Region I und III sind asymptotisch flache Regionen, II ist das Innere des Schwarzen Lochs und IV ist ein weißes Loch. Die fetten Hyperbeln in den Regionen II und IV sind die Singularitäten. Die Diagonalen durch den Ursprung sind die Ereignishorizonte. Der Ursprung (eigentlich eine 2-Sphäre mit unterdrückten Winkelkoordinaten) ist der Schlund eines nicht passierbaren Wurmlochs, das die getrennten "Universen" I und III verbindet. Radiale Lichtstrahlen bleiben 45-Grad-Diagonallinien im Kruskal-Szekeres-Diagramm. Die gestrichelten Hyperbeln sind Linien konstanten Schwarzschilds r Koordinate, und die gestrichelten radialen Strahlen sind konstante Linien t . Sie können sehen, wie der Ereignishorizont wo zu einer Koordinaten-Singularität wird r und t Rollen tauschen.

Wenn Sie nun eine Weltlinie von Region I in Region II ziehen, wird deutlich, dass sie den Horizont in endlicher Eigenzeit überquert, und, was noch wichtiger ist, der vergangene Lichtkegel des Ereignisses, wo er auf die Singularität trifft, kann unmöglich die gesamte Raumzeit enthalten. Die kurze Antwort auf Ihre Frage lautet also nein , jemand, der in ein schwarzes Loch fällt, sieht nicht das Ende des Universums. Ich kenne die Formel nicht, nach der du fragst T , aber im Prinzip können Sie es von Lichtstrahlen auf dem Diagramm ablesen und einfach in die gewünschte Koordinate / Eigenzeit umwandeln.

@JohnRennie Ich sehe jetzt, dass Sie bei der vorherigen Frage dieses Argument so ziemlich selbst vorgebracht haben (und natürlich vermutete ich, dass Sie bereits davon gehört haben), es aber irgendwie unbefriedigend zu finden scheinen. Aber ich muss gestehen, dass ich nicht sehe, was daran nicht überzeugend ist, genauso wenig wie zu sagen, dass die Geometrie am Nordpol gut funktioniert, obwohl einige gemeinsame Koordinatensysteme einen anderen Eindruck vermitteln ...
Ah, ja, ich sehe es jetzt und es ist ein so klares Argument :-) Alle radialen Null-Geodäten verlaufen von rechts unten nach links oben, und wo immer Ihr einfallender Beobachter auf die Singularität trifft, trifft jeder Lichtstrahl die Singularität rechts oben vom Beobachter können von ihnen nicht gesehen werden. Was fehlt, ist, wie man die Flugbahn des einfallenden Beobachters konstruiert. Gibt es einfache Regeln, die mir helfen, den (zeitähnlichen) Pfad des Beobachters in Ihrem Diagramm zu zeichnen?
Richtig, ich habe Ihre Antwort umgeschrieben, damit ich sie besser verstehe. Könnten Sie sich meine Umschreibung ansehen und gegebenenfalls kritisieren (offensichtlich ist Ihre Antwort die akzeptierte! :-)).
Ich habe einen kleinen, aber wichtigen Einwand gegen dieses Argument (cc @John Rennie): Wenn unsere Koordinaten versehentlich einen Punkt (in 1 + 1D) zu einer Linie erweitert haben, bedeutet das Treffen verschiedener Teile der Linie nicht, dass die Trajektorien dies nicht tun konvergieren. Betrachten Sie Karten der Erde, die den Nordpol zu einer horizontalen Linie erweitern - Sie würden nicht sagen, dass jemand bei (90 N, 20 E) von jemandem bei (90 N, 130 W) getrennt ist, obwohl das Diagramm dies impliziert. Ebenso der Punkt, an dem sich meine Weltlinie schneidet u = v sieht disjunkt aus, wo ein Nullstrahl diese Linie schneidet, aber Sie müssen beweisen , dass sie unterschiedlich sind.
@JohnRennie Ich helfe gerne. :)
@ChrisWhite: Guter Punkt. Meinst du den Horizont oder die Singularität? Für den Horizont sind es die Schwarzschild-Koordinaten, die analog zu Ihrer verzerrten Karte sind, nicht die Kruskal-Koordinaten. Andererseits ist die Singularität eine echte physikalische Singularität, keine bloße Koordinaten-Singularität, also weiß ich nicht wirklich, wie ich darüber sprechen soll.
Aus diesem Diagramm ist ersichtlich, dass sich sowohl der weit entfernte Beobachter als auch der einfallende Beobachter zu einem beliebigen Zeitpunkt in der Zukunft immer noch außerhalb des Schwarzen Lochs befinden (es gibt eine zeitgleiche gestrichelte Linie, die die beiden verbindet). Da das Schwarze Loch in endlicher Zeit verdunstet, bedeutet dies, dass der einfallende Beobachter niemals den Ereignishorizont überschreiten wird. Aber ja, der einfallende Beobachter in endlicher Eigenzeit überquert entweder den Horizont (wenn es keine Hawking-Strahlung gibt) oder sieht das BH verschwinden.

Dies ist eine Umschreibung von Michael Browns Antwort, um mir zu helfen, meine Gedanken klar zu bekommen, und möglicherweise auch allen anderen, die daran interessiert sind, zu helfen, ihre Gedanken klar zu bekommen :-) Michael präsentiert eine sehr einfache Antwort auf meine Frage, die auf der Geometrie der Raumzeit basiert um das Schwarze Loch.

Der entscheidende Punkt ist, dass die üblichen Radius/Zeit-Schwarzschild-Koordinaten nicht hilfreich sind, weil sie verschleiern, was vor sich geht. Um dies zu umgehen, verwenden wir eine Koordinatentransformation, um die Raumzeit um das Schwarze Loch unter Verwendung der Kruskal-Szekeres- Koordinaten zu zeichnen u und v . So sieht das Ergebnis aus:

Schwarzes Loch

Das u Koordinate ist horizontal und die v Koordinate ist vertikal.

Das Problem mit diesen Koordinaten ist, dass sie höchst unintuitiv sind. Eine Verschiebung in u oder v entspricht keiner einfachen physikalischen Größe, im Gegensatz zu einer Verschiebung in der üblichen radialen Koordinate r oder Zeitkoordinate t . Trotzdem vereinfachen die KS-Koordinaten die Dinge drastisch wie folgt:

In diesen Koordinaten konstant r ist eine Hyperbel, wie durch die gestrichelte Linie gezeigt. Der Ereignishorizont ist die durchgezogene 45°-Linie. Sie können so denken t steigt, wenn Sie sich nach oben bewegen - das tut es, wenn auch nicht linear. Die Singularität ist die rote Hyperbel (dies ist ein Raumzeitdiagramm, erinnern Sie sich, also ist die Singularität eine Kurve, kein Punkt). Die Region habe ich beschriftet ich ist das Äußere des Schwarzen Lochs und der Region, die ich markiert habe ich ich ist die Region innerhalb des Ereignishorizonts. Ignorieren Sie den Bereich des Diagramms unten links, da er für meine Frage nicht relevant ist.

Schließlich ist das Schlüsselmerkmal, das es ermöglicht, meine Frage zu beantworten, dass alle radial einfallenden Lichtstrahlen gerade 45 ° -Linien sind, die von rechts unten nach links oben verlaufen. Ich habe mehrere solcher Lichtstrahlen als magentafarbene Linien gezeichnet.

Jetzt können wir meine Frage beantworten. Wir beginnen mit einer Rakete, die in konstantem Abstand vom Schwarzen Loch entfernt schwebt, was durch die schwarz gestrichelte Hyperbel der Konstante dargestellt wird r (Wie ich oben erwähnt habe, können Sie sich vorstellen, dass die Zeit zunimmt, wenn Sie nach oben gehen). Zum Zeitpunkt t 0 Unser Beobachter verlässt die Rakete und beginnt, auf das Schwarze Loch zuzufallen. Die blaue Linie zeigt die Flugbahn, der der Beobachter folgt. Der Betrachter trifft die Singularität an dem Punkt, an dem sich die blaue und die rote Linie treffen.

Zum Zeitpunkt t 1 die Rakete richtet einen Lichtstrahl auf den einfallenden Beobachter. Der unter 45° laufende Lichtstrahl erreicht den Betrachter, bevor er den Ereignishorizont überquert – so weit, so gut. Zum Zeitpunkt t 2 die Rakete strahlt einen zweiten Lichtstrahl auf den Beobachter, und dieser Lichtstrahl erreicht den Beobachter genau dann, wenn er auf die Singularität trifft. Zum Zeitpunkt t 3 die Rakete strahlt einen dritten Lichtstrahl in das Schwarze Loch, der den Beobachter aber nicht erreicht, weil der Beobachter die Singularität bereits getroffen hat und nicht mehr existiert. Das heißt, der Betrachter sieht den gerade freigesetzten Lichtstrahl nie t 3 . Der Beobachter sieht jeden Lichtstrahl, der dazwischen freigesetzt wird t 0 und t 2 , sieht aber keinen Lichtstrahl, der danach freigesetzt wird t 2 . Die gestrichelte magentafarbene Linie markiert also die Grenze zwischen Lichtstrahlen, die der Beobachter sehen kann, und solchen, die er nicht sehen kann.

Und da ist die Antwort auf meine Frage. Der Beobachter sieht das Ende des Universums nicht, denn der letzte Lichtstrahl, den er sieht, ist der, der zu diesem Zeitpunkt freigesetzt wird t 2 .

Dies gibt mir keine einfache Möglichkeit, den Wert von zu berechnen t 2 , weil ich einen Ausdruck für die Flugbahn des einfallenden Beobachters (blaue Linie) herleiten müsste und das ist schwierig. Trotzdem zeigt sich das t 2 ist endlich, also mit der Notation in meiner Frage, T ist begrenzt.

Nebenbei und im Zusammenhang mit der Interpretation der u- (raumähnlichen) und v- (zeitähnlichen ) Koordinaten ist zu beachten, dass im Außenbereich diagonale Linien durch den Ursprung Linien mit konstanter Schwarzschild-Zeitkoordinate t sind . Und wie Sie betonen, ist der Ort der konstanten Schwarzschild-Raumkoordinate r eine Hyperbel. Sehen Sie sich nun mit diesem Bild die Rindler-Koordinaten an.
Sieht gut aus!....
John Rennie, Michael Brown, ich grüße euch beide und verneige mich in tiefem Respekt! Das ist entzückend und genau die Art von diagrammartiger Klarheit dessen, was passiert, auf die ich am meisten gehofft hatte, aber Angst hatte zu fragen (die genauen Zahlen sind zweitrangig; es ist der konzeptionelle Rahmen, der entscheidend ist). Ich werde vielleicht noch mehr Kommentare haben, nachdem ich diesen hier ausgearbeitet habe (er verdient eine genaue Prüfung!), aber hauptsächlich möchte ich nur Danke sagen. Ich bezweifle, dass ich der einzige bin, der sich im Laufe der Jahre darüber geärgert hat (und es gibt statistische Beweise, die diese Behauptung stützen!). Außerdem, John Rennie, danke für diese schöne endgültige Zahl. Sehr deutlich!
Sie könnten auch an Gullstrand-Painlevé-Koordinaten interessiert sein , die eine klarere physikalische Interpretation als Kruskal-Szekeres haben, da sie an einen radial frei fallenden Beobachter angepasst sind, aber am Horizont regelmäßig bleiben. Der zu zahlende Preis liegt außerhalb der Diagonalen in der Metrik.
Michael Brown, danke, eine weitere nette Referenz. Obwohl ich immer noch keine Zeit hatte, dieses KS-System zu studieren, war ich ziemlich überrascht, wie vertraut es den Zahlen erschien, mit denen ich für mein eigenes Verständnis von SR gespielt habe ... was mich vermuten lässt, dass ich etwas schlecht interpretiere ? ZB können SR-Lichtkegel sicherlich nicht direkt auf die gekrümmte Raumgeometrie der Region in der Nähe eines Schwarzen Lochs abgebildet werden, oder? Und ich habe mir die GP-Koordinaten nicht wirklich über einen kurzen Blick hinaus angesehen. Hoffentlich habe ich bald mehr Zeit...
@TerryBollinger: siehe auch physical.stackexchange.com/questions/28297/… wo ich die GP-Koordinaten verwende, um zu zeigen, warum Licht nicht aus einem schwarzen Loch entkommen kann.
@TerryBollinger Es ist eine allgemeine Tatsache der Differentialgeometrie, dass jeder zweidimensionale Raum (lokal) konform flach ist. Wenn Sie also die Winkel unterdrücken und nur die betrachten r t Raum finden Sie immer eine Koordinatentransformation, die (radiale) Nullstrahlen zu 45-Grad-Diagonalen macht, und die Metrik in die Form bringen ω 2 ( u , v ) ( d v 2 + d u 2 ) + r 2 ( u , v ) d Ω 2 2 für einige geeignete Funktionen ω ( u , v ) und r ( u , v ) . Radiale Lichtstrahlen werden v = ± u + konst . Du kannst nicht immer den gleichen Trick machen > 2 Dimensionen - zu viele Freiheitsgrade in der Metrik.
Danke, das ist nützlich; und sogar schematisch kann ich sehen, wie die richtigen Manipulationen das Schöne neu erschaffen könnten 45 Lichtwege von SR. Mein Punkt intensiver Neugier ist, klar zu verstehen, wie die u und v Koordinaten spielen sich in Bezug auf die relative Zeitdilatation (Zeiteinheitsverhältnisse) ab. Dieses Wochenende keine Zeit, aber hoffentlich bald...
Mir ist nicht klar, dass dies der Weg ist, den der einfallende Beobachter nehmen würde. Können Sie anhand dieses Koordinatensystems eine Intuition über die verstreichende Zeitskala geben? Ich denke, das war der Kern der Frage zu Beginn. Ist es zum Beispiel klar, dass der Pfad nicht auf 45 Grad asymptot, bevor er die Singularität berührt?
@Inverse: Ich denke, meine Illustration des Pfads ist etwas daneben und sollte eher wie meine Antwort auf diese Frage aussehen
Eine Formel für t 2 t 0 ist in meiner Antwort angegeben.
@inverse Ich denke , dass der Ort für ein einfallendes massives Teilchen asymptotisch zu einer Linie ist, die den Ursprung mit der Stelle verbindet, an der das Teilchen auf die Singularität trifft. Dies ist natürlich eine Linie mit konstantem Schwarzschild t .
Es ist in diesen Koordinaten schwer zu erkennen, aber tatsächlich liegt der Zeitpunkt, an dem der Beobachter den Horizont erreicht, in der unendlichen Zukunft. Sie können sich vorstellen, dass die zeitgleichen Linien ("Isotemps"), bei denen es sich um gerade Linien handelt, die den Ursprung kreuzen, dichter werden, wenn man sich der Nulldiagonalen (Horizont) nähert. Als solcher wird der Beobachter niemals den Horizont überqueren, er müsste unendlich viele Isotemps überqueren (aber er wird auch nicht das Ende des Universums sehen).
Übrigens ist dieses Koordinatensystem nur eine einfache Split-Complex-Ebene. Zeit ist Split-Complex-Argument und Radius ist Split-Complex-Modulus. Die Zeit zwischen zwei Isotemps ist proportional zur Fläche des parabolischen Sektors zwischen diesen beiden Linien.
Interessanterweise wird die Zeit, in der der Beobachter den Ereignishorizont erreicht, zwar unendlich, aber zur Euler-Mascheroni-Konstante regularisiert γ mathoverflow.net/questions/389694/…
Das Split-Complex-Modell zeigt, dass das wahre Zentrum des Schwarzen Lochs sein Ereignishorizont ist (die Abstände zwischen allen Punkten auf den Nulldiagonalen sind Null, es ist eine Kugel mit Nullradius). Die „Singularität“ befindet sich nicht im Zentrum des Schwarzen Lochs, es ist eine kugelförmige Oberfläche mit imaginärem Radius in konstantem Abstand zum Horizont. An Singularität gibt es nichts „Singulares“, aber die wirkliche Singularität befindet sich am Horizont: Alle Punkte dort entsprechen singulären Matrizen.
Sind Sie sicher, dass Ihr Beobachter das bei t2 emittierte Photon sehen wird? Die Diagonale, die von links unten nach rechts oben verläuft, ist beschriftet
t =
auf Kruskal-Szekeres Diaram. Bedeutet das nicht, dass sie unendlich viel Zeit brauchen, um sich zu treffen? Und wenn das der Fall ist, ist das wahrscheinlich letzte Photon, das der Beobachter sieht, dasjenige, das zum Zeitpunkt zwischen t1 und t2 emittiert wurde und den Beobachter trifft, kurz bevor er den Ereignishorizont überquert.
@KamilSzot Das t = bezieht sich auf die Schwarzschild-Koordinatenzeit, nicht auf die vom einfallenden Beobachter gemessene Zeit.
Die Schwarzschild-Koordinatenzeit kann also gleich sein und es ist eine normale Sache, nicht das Ende der Zeit aus der Sicht aller weit draußen?

Die derzeit akzeptierte Antwort umgeht die Frage nach der Berechnung, welche Ereignisse tatsächlich mit Schwarzschild-Koordinaten zu sehen sind. Diese Frage lässt sich mit Schwarzschild-Koordinaten sowohl numerisch als auch analytisch beantworten . Die Antwort ist natürlich, dass der vergangene Lichtkegel für den Grenzfall nicht das gesamte Universum außerhalb des Schwarzen Lochs umfasst und dass eine endliche Zeit zur Verfügung steht, um einem fallenden Objekt (auch in Schwarzschild-Koordinaten) ein Signal zu geben, das davon abhängt, wo das ist fallender Beobachter wurde befreit.

Es gibt zwei separate Probleme mit jeweils zwei separaten Fällen. Die erste besteht darin, herauszufinden, ob Licht einen fallenden Beobachter abfängt, bevor er den Ereignishorizont erreicht. Allerdings muss dann eine kleine zusätzliche Korrektur vorgenommen werden, um herauszufinden, ob ein Lichtsignal einen fallenden Beobachter noch abfangen kann, nachdem er den Ereignishorizont überschritten hat, aber bevor er die Singularität erreicht.

1. Ob Licht ein Objekt abfangen kann, bevor es den Ereignishorizont erreicht

(a) Objekt, das aus der Unendlichkeit fällt

Ich beginne mit einem Beobachter in einem Radius r 0 (Alle Radien werden als Vielfache des Schwarzschild-Radius ausgedrückt r s ). Der Beobachter wird zur Zeit übergeben t 0 (in Schwarzschild-Koordinaten, was gleich ist τ = 0 nach der eigenen Uhr des Beobachters), durch ein Objekt, das aus der Unendlichkeit radial nach innen in Richtung des Schwarzen Lochs fällt (wo es in Ruhe begann). Manchmal Δ t später schießt der Beobachter einen Laserstrahl radial nach innen. Das Problem ist, das Maximum herauszuarbeiten Δ t das wird das fallende Objekt abfangen und dieses dann in a umwandeln Δ τ in Bezug auf die Eigenzeit nach Ansicht des Beobachters. Dass es ein Maximum geben muss Δ t und Δ τ wird konzeptionell leicht durch Berücksichtigung (zB) von Kruskal-Szekeres-Koordinaten erstellt.

Die Null-Geodäte (in Schwarzschild-Koordinaten), der das nach innen wandernde Licht folgt (in c = 1 Einheiten) ist:

(1) t = r r s ln | r r s r 0 r s | + a + Δ t ,
wo die Konstante a = r 0 + t 0 .

Die Geodäte, gefolgt von einem Körper, der in Ruhe aus der Unendlichkeit entlassen wird, ist (z. B. siehe Gl. 25.38 im Abschnitt „Orbits of Particles“ von „Gravitation“ von Misner, Thorne & Wheeler, 2017, Princeton University Press)

(2) t = r s ( 2 3 ( r r s ) 3 / 2 2 ( r r s ) 1 / 2 + ln | r / r s + 1 r / r s 1 | ) + b
Die Konstante b kann gewählt werden, um sicherzustellen, dass das Objekt den Punkt passiert ( t 0 , r 0 ) - daher:
(3) b = t 0 r s ( 2 3 ( r 0 r s ) 3 / 2 2 ( r 0 r s ) 1 / 2 + ln | r 0 / r s + 1 r 0 / r s 1 | )

Indem ich diese Geodäten aufzeichnete und eine Halbierungsmethode verwendete, um zu bestimmen, wann und ob sie sich schneiden, konnte ich das Maximum bestimmen Δ t ( T im OP, obwohl ich mein Objekt im freien Fall aus der Unendlichkeit gestartet habe), die es dem Licht immer noch ermöglicht, das fallende Objekt abzufangen, abhängig davon, woher dieses Licht emittiert wird. Das Ergebnis scheint stabil zu sein, um die Toleranz zu reduzieren (ich verwendete 10 14 r s ).Maximale Zeitverzögerung, freier Fall aus der Unendlichkeit

Ein Beispiel für den Grenzfall ist unten gezeigt. Die rote Kurve ist die leichte Geodäte, während die blaue Kurve die Geodäte eines Objekts zeigt, das aus der Unendlichkeit fällt und (in diesem Fall) hindurchgeht. 5.8 r s bei t = 0 . Nur Ereignisse unterhalb der roten Kurve könnten von einem fallenden Beobachter gesehen werden.Grenzfall für den freien Fall aus dem Unendlichen

Diese Kurve habe ich dann analytisch "abgeleitet". Gleichung (1) umstellend können wir schreiben

r r s = ( r 0 r s ) exp ( ( a + Δ t r ) / r s ) exp ( t / r s )
und wenn (nahe der Grenze, wo Licht das fallende Objekt abfangen kann) lassen wir t dann groß werden r r s und wir können schreiben
(4) r r s ( r 0 r s ) exp ( ( a + Δ t r s ) / r s ) exp ( t / r s ) ,
wo wir diese Tatsache ausnutzen, dass die Grenze von r exp ( r / r s ) wie r r s ist nur r / e .

Wenn wir Gleichung (2) auf ähnliche Weise umstellen, erhalten wir

r / r s 1 r / r s + 1 = exp ( t / r s ) exp ( 2 3 ( r r s ) 3 / 2 2 ( r r s ) 1 / 2 + b r s ) .
Auch hier argumentieren wir um den Grenzfall herum r r s und so können wir schreiben
r / r s = 1 + 2 exp ( b / r s 8 / 3 ) exp ( t / r s )
Quadrieren Sie dies und vernachlässigen Sie die exp ( 2 t / r s ) Begriff:
(5) r r s 4 r s exp ( b / r s 8 / 3 ) exp ( t / r s ) )

Ob es einen Schnittpunkt gibt oder nicht, wird dadurch bestimmt, ob das Verhältnis der Gleichungen (4) und (5) kleiner als 1 ist t .

lim t ( r 0 r s ) exp ( ( a + Δ t r s ) / r s ) exp ( t / r s ) r s ( 1 + 4 exp ( b / r s 8 / 3 ) exp ( t / r s ) ) < 1
was dazu führt
( r 0 r s ) exp ( ( a + Δ t r s ) / r s ) 4 r s exp ( b / r s 8 / 3 ) < 1
exp ( Δ t / r s ) < 4 r s r 0 r s exp ( b a r s 5 3 )
Δ t < ln ( 4 r s r 0 r s ) r s + ( b a r s 5 3 ) r s
Wiedereinfügen der Ausdrücke für a und b
Δ t < ln ( 4 r s r 0 r s ) r s + ( 2 3 ( r 0 r s ) 3 / 2 + 2 ( r 0 r s ) 1 / 2 ln | r 0 / r s + 1 r 0 / r s 1 | 5 3 ) r s r 0
Dies stimmt mit dem überein, was oben aufgetragen ist.

Um daraus ein maximales Eigenzeitintervall zu machen Δ τ aus Sicht des Betrachters würde das Ergebnis multipliziert werden ( 1 r s / r 0 ) 1 / 2 .

(b) Objekt fällt aus der Ruhe bei t 0 , r 0

Nun ist der Aufbau so, dass der Beobachter das Objekt loslässt t 0 , r 0 , wartet dann ein (koordiniertes) Zeitintervall Δ t vor der Signalisierung.

Gleichung (1) ist in diesem Szenario immer noch gültig, jedoch muss Gleichung (2) durch die folgende Geodäte für ein Objekt ersetzt werden, das frei aus der Ruhe fällt t 0 , r 0 .

(6) t t 0 r s = ln | ( r 0 / r s 1 ) 1 / 2 + bräunen ( η / 2 ) ( r 0 / r s 1 ) 1 / 2 bräunen ( η / 2 ) | + ( r 0 r s 1 ) 1 / 2 ( η + r 0 2 r s ( η + Sünde η ) ) .
Hier der "Zykloidenparameter" η ( r ) ist definiert durch
r = r 0 2 ( 1 + cos η )

Wie r r s , wächst der erste Term in Gleichung (6) exponentiell, während der zweite Term, den ich als definieren werde b ( r ) / r s , tendiert zu einer Konstante:

lim r r s b ( r ) = b r s = r s ( r 0 r s 1 ) 1 / 2 ( η r s + r 0 2 r s ( η r s + Sünde η r s ) ) ,
wo
cos η r s = ( 2 r s r 0 1 ) .

Mit der Identität that bräunen η / 2 = Sünde η / ( 1 + cos η ) , dann

bräunen ( η / 2 ) = ( r 0 r 1 ) 1 / 2 .
Indem wir dies in Gleichung (6) einsetzen, können wir festlegen t 0 = 0 , potenzieren und finden
( r 0 r s 1 ) 1 / 2 ( 1 exp [ b t r s ] ) = ( r 0 r 1 ) 1 / 2 ( 1 + exp [ b t r s ] )
Quadrieren Sie dies und vernachlässigen Sie enthaltende Terme exp ( 2 t / r s ) wie t groß wird, kann dies neu angeordnet werden, um zu geben
r = r s ( 1 + 2 exp [ ( b t ) / r s ] ) 1 2 exp [ ( b t ) / r s ] + ( 4 r s / r 0 ) exp [ ( b t ) / r s ] .
Auch hier suchen wir nach einem einschränkenden Verhalten im Allgemeinen t , dann kann der Nenner als Binomial entwickelt werden, wobei nur die ersten beiden Terme beibehalten werden. Multiplikation mit dem Zähler ergibt dann:
(7) r r s 4 r s ( 1 r s r 0 ) exp [ b t r s ] .

Die Begrenzung finden Δ t für die ein Lichtstrahl vom Beobachter das fallende Objekt "einfängt", nehmen wir das Verhältnis der Gleichungen 4 und 7, gesetzt b = b r s und fordere, dass dies kleiner als 1 ist. Dies ergibt

exp [ Δ t r s ] < 4 ( r s r 0 ) exp [ b r s r s ] exp [ r s r 0 r s ]
und daher
Δ t < r s ln ( 4 r s r 0 ) + b r s + r s r 0

Das Ergebnis ist unten als rote Kurve aufgetragen (und ich habe bestätigt, dass es mit einer numerischen Halbierungsmethode korrekt ist) und mit Fall 1 mit dem frei fallenden Objekt aus der Unendlichkeit verglichen (blaue Kurve, wie im ersten Bild). Wie erwartet das Erlaubte Δ t ist größer, wenn das Objekt aus der Ruhe entlassen wird.

Wie zuvor ist dieses Ergebnis das maximale Schwarzschild-Koordinatenzeitintervall. Sie muss um den entsprechenden Zeitdilatationsfaktor reduziert werden ( 1 r s / r 0 ) 1 / 2 um das maximale richtige Zeitintervall zu erhalten.Maximale Zeitverzögerung für das Herabfallen eines Objekts aus der Ruhe

Ein Beispiel für den Grenzfall ist unten gezeigt. Die rote Kurve ist die Geodäte des Lichts, die blaue Kurve ist die Geodäte des fallenden Objekts. Nur Ereignisse unterhalb der roten Kurve (die einem Gradienten von -1 entspricht) können von einem Objekt "gesehen" werden, das aus der Ruhe in ein Schwarzes Loch fällt, von (in diesem Fall) ungefähr 5.8 r s .Beispiel für den Grenzfall.

2. Ob Licht ein Objekt abfangen kann, bevor es die Singularität erreicht

Die obige Antwort gibt die maximale (Koordinaten-) Zeitverzögerung für ein Signal von einem stationären Beobachter an, um ein fallendes Objekt zu erreichen, bevor es den Ereignishorizont erreicht. ( Δ t ) E H . Aber das beantwortet die (Überschrift-)Frage nicht vollständig, weil das Objekt während der Zeit, die es braucht, um die Singularität zu erreichen, nachdem es den Ereignishorizont überschritten hat, immer noch Licht empfangen kann. Dies ist am deutlichsten in Kruskal-Szekeres-Koordinaten zu sehen, aber auch hier ist es möglich , dies (ziemlich einfach) in Schwarzschild-Koordinaten zu lösen.

Bedingung hierbei ist, dass die Koordinatenzeit der verzögerten Lichtgeodätischen kleiner oder gleich der Koordinatenzeit der fallenden Objektgeodäten sein muss r = 0 .

Diese Bedingung ist eigentlich ziemlich einfach zu finden. Für den Fall, dass das Objekt frei aus der Unendlichkeit fällt, zeigen die Gleichungen (1-3), dass das Original Δ t dass ich abgeleitet sollte als erhöht werden

( Δ t ) s ich n g u l a r ich t j = r s ln ( r s r 0 r s ) r s ( 2 3 ( r 0 r s ) 3 / 2 2 ( r 0 r s ) 1 / 2 + ln | r 0 / r s + 1 r 0 / r s 1 | ) r 0
Oder in Bezug auf das vorherige Ergebnis.
( Δ t ) s ich n g u l a r ich t j = ( Δ t ) E H + r s ( 5 3 2 ln 2 ) = ( Δ t ) E H + 0,280 r s

Für den Fall, dass ein Objekt aus der Ruhe fällt, sehen wir das η = π bei r = 0 , so dass, wenn die Koordinatenzeit kleiner oder gleich der Koordinatenzeit des Objekts sein soll r = 0 ergibt sich aus den Gleichungen (1) und (6) als

( Δ t ) s ich n g u l a r ich t j = r s ln ( r s r 0 r s ) + π r s ( r 0 r s 1 ) 1 / 2 ( 1 + r 0 2 r s ) r 0 ,
was größer ist als ( Δ t ) E H um einen Betrag, der davon abhängt r 0 , ist aber asymptotisch zu den aus dem Unendlichen fallenden Ergebnissen als r 0 groß wird. Diese neue Beziehung ist unten dargestellt – die höhere rote Kurve ist die maximale (Koordinatenzeit-)Verzögerung, die toleriert werden kann und dennoch ein Signal sendet, das das fallende Objekt vor der Singularität erreicht. Das untere Diagramm zeigt die Differenz zwischen diesem und dem vorherigen Ergebnis für die Verzögerung, um das Objekt noch vor dem Ereignishorizont zu erreichen.Maximale Verzögerung vor der Singularität

Die folgende Grafik sollte die Dinge klarer machen. Es zeigt die Geodäten auf beiden Seiten oder r s im Falle eines herunterfallenden Gegenstandes r = 2 r s bei t = 0 . Die leichte Geodäte in Rot ist zum einen so berechnet, dass sie das Objekt gerade so abfängt r r s und hat ( Δ t ) E H = 3.834 r s / c . Aber wir sehen, dass diese Geodäte das fallende Objekt „überholt“, bevor es die Singularität bei erreicht r = 0 . Allerdings grünes Licht geodätisch, mit ( Δ t ) s ich n g u l a r ich t j = 4.283 r s / c schneidet das Objekt geodätisch genau ab r = 0 .Geodäten auf beiden Seiten von r_s

Ich hatte auch daran gearbeitet, aber du bist mir zuvorgekommen. Ich schätze du bist das Photon und ich bin das sich langsam bewegende Objekt :) Ich habe gerade meine eigene Version gepostet.
Ich glaube, du hast ein Extra verpasst r 0 in deiner letzten Gleichung. Seltsamerweise scheint Abb. 31.4 in MTW einen Fehler zu enthalten: die u -Koordinate des Weges F F sollte eher steigen als sinken. Bei der Singularität, d u / d v = u / v .
@Pulsar Es ist teilweise diese Unsicherheit, die mich davon abgehalten hat, die (hinzugefügten) Ergebnisse oben in KS-Koordinaten zu überprüfen. Bist du dir sicher? Muss die Flugbahn nicht abnehmen r und abnehmend t unter dem Horizont, mit einer zeitähnlichen Geodäte, die die Linien der Konstante schneidet t im rechten Winkel, wenn es die Singularität erreicht?
Ich bin ziemlich sicher. Ich habe diesen Artikel gefunden , der mit meinen Berechnungen übereinstimmt. Schauen Sie sich Abb. 2 und Gleichung 23 an.
@Pulsar Ich stimme zu (und war verwirrt), dass eine zeitähnliche Geodäte asymptotisch zu einer Konstantenlinie enden sollte t ? Sowohl AA'' als auch FF'' sehen in diesem MTW-Diagramm falsch aus.
Richtig. Offensichtlich hat sogar die Bibel ein paar Fehler.
Sehr schön, aber ich zuckte jedes Mal zusammen, wenn ich las, dass Sie die Halbierung verwendet haben (aber um fair zu sein, ich habe die letzten Tage damit verbracht, die Wurzelfindungsalgos bei der Arbeit zu verbessern).
@KyleKanos Wenn es länger gedauert hätte, als ich zum Bewegen der Maus brauche, hätte ich vielleicht etwas Zeit in etwas Komplexeres investiert.
@RobJeffries fair genug.

Ich stimme zu, dass für eine Raumzeit, die genau Schwarzschild ist, der einfallende Beobachter nicht die gesamte Geschichte des Universums sieht. Es stellt sich jedoch heraus, dass dies nicht der allgemeine Fall ist, den man für ein astrophysikalisches Schwarzes Loch erwarten würde, das durch den Kollaps einer annähernd kugelförmigen Verteilung von Materie entstanden ist. Dieses Thema wird tatsächlich aktiv erforscht, und es gibt einige sehr interessante Ergebnisse darüber, wie das Innere eines Schwarzen Lochs tatsächlich aussieht. Siehe zum Beispiel dieses kürzlich erschienene Papier .

Der Grund, warum der einfallende Beobachter in Schwarzschild nicht die gesamte Geschichte des Universums sieht, ist, dass die Singularität raumartig ist. Dies bedeutet, dass es eine Reihe von Punkten gibt, an denen der einfallende Beobachter die Singularität treffen kann, und jeder Punkt kann nur einen Teil des Universums in seiner kausalen Vergangenheit sehen.

Aber die Menschen kennen seit langem andere Arten von Schwarzen Löchern, die dieses Verhalten nicht teilen. Die bekanntesten Beispiele sind die Reissner-Nordstrom-Lösung für ein geladenes, kugelsymmetrisches Schwarzes Loch und die Kerr-Lösung für ein rotierendes Schwarzes Loch. Diese beiden haben zeitähnliche Singularitäten, und daher ist die Situation ganz anders. Hier ist ein Kausaldiagramm eines Reissner-Norstrom-Schwarzen Lochs:

RN BH Kausaldiagramm

Die vertikalen gezackten Linien repräsentieren die zeitlichen Singularitäten dieses Schwarzen Lochs. In diesem Fall ist es möglich, die Singularität zu vermeiden und in ein neues Universum aufzutauchen, das Sie oben auf diesem Bild anbringen könnten. In diesem Fall sollten Sie beim Überqueren des inneren Horizonts zurückblicken und die gesamte Geschichte oder das Universum sehen können.

Dies wirft jedoch einen problematischen Punkt auf. Der Beobachter passiert den inneren Horizont in endlicher Eigenzeit und kann dennoch alles Licht sehen, das aus der gesamten unendlichen Geschichte des Universums in das Schwarze Loch eintritt. Da Licht Energie hat, könnten Sie denken, dass diese Anhäufung von Strahlung aus dem äußeren Universum zu einer starken Krümmung führen sollte, und das tut es tatsächlich. Dies ist als Masseninflation des Schwarzen Lochs bekannt. Kerr-Schwarze Löcher teilen diese Eigenschaft, obwohl die Struktur der Singularität in diesem Fall komplizierter ist.

Für generische Schwarze Löcher, die nicht genau Schwarzschild sind, wird also ein anderes Verhalten erwartet. Die Störungen neigen dazu, die Singularität von raumartig zu einem Verhalten wie eine Nulloberfläche zu ändern, dh den Lichtbahnen zu folgen. Ein Bild aus dem obigen Papier zeigt diese Situation:

Null Horizont

Das äußere Universum lebt im unteren rechten Dreieck dieses Bildes. Die Zeilen beschriftet C H + sind die Nullsingularitäten. Das Papier stellte fest, dass diese Situation aus der Störung der Schwarzschild-Lösung mit Skalarfeld-Materie resultierte. In diesem Fall würden Sie, wenn Sie vom äußeren Universum in das Schwarze Loch fallen, auf die Null-Singularitäten stoßen, und wenn Sie die rechte treffen, werden Sie die gesamte Geschichte des Universums in dem Sinne sehen, dass all das Sie werden Zugang zu Licht haben, das aus beliebig späten Zeiten der Geschichte des Universums in das Schwarze Loch eintritt.

Meinen Sie, dass Sie, wenn Sie ein masseloser Punkt sind und frei in ein geladenes oder rotierendes Schwarzes Loch fallen, tatsächlich unendlich viel Zeit brauchen, um die Singularität zu erreichen, aber da Sie immer schneller durch die Raumzeit beschleunigen, verlangsamt sich Ihre Zeit genug, um nur eine endliche Menge zu passieren?

(Die Antwort von Michael Brown ist die richtige Antwort und dient lediglich zur Verstärkung durch ein hinzugefügtes Diagramm.)

Unten ist Abbildung 31.4 von Seite 835 von Gravitation (MTW).

Geben Sie hier die Bildbeschreibung ein

Beide Diagramme sind von der Schwarzschild-Geometrie. Beachten Sie, dass in den Kruskal-Szekeres-Koordinaten Lichtkegel erscheinen, wie sie es in der Minkowski-Raumzeit tun.

Wie Michael betont, sind lichtähnliche radiale Geodäten 45-Grad-Linien, wie man sehen kann, wenn man die Geodäte B betrachtet.

Es gibt eindeutig lichtähnliche Weltlinien, die den Horizont nach einigen zeitähnlichen Weltlinien kreuzen, sodass die Weltlinie eines Astronauten, der radial auf das Loch zufällt, nicht alle lichtähnlichen radialen Weltlinien schneidet, bevor er den Horizont überquert.

Es ist auch klar, dass es lichtähnliche Weltlinien gibt, die nach einigen zeitähnlichen Weltlinien auf der Singularität enden.

Daher sieht der Astronaut nicht die unendliche Zukunft, bevor er den Horizont überquert oder der Singularität begegnet.

Außerdem, und das ist nur eine interessante Nebenbemerkung, ist die Schwarzschild-Lösung die kugelsymmetrische , statische (zumindest außerhalb des Horizonts) Lösung der Einstein-Gleichungen. Mit anderen Worten, es gibt kein „Ende des Universums“ in dieser Lösung .

Ah, wir haben gleichzeitig Ausarbeitungen von Michaels Antwort geschrieben :-) Könnten Sie sich meine Version ansehen und gegebenenfalls kritisieren?
Einige von uns denken, dass die zeitähnlichen Geodäten von MTW in diesem Diagramm möglicherweise falsch in KS-Koordinaten skizziert sind (AA'' und insbesondere FF''). Hätten Sie dazu eine Meinung?
Die Diagonale, die von links unten nach rechts oben verläuft, ist beschriftet
t =
Wieso ist das nicht das Ende des Universums?
@KamilSzot, diese Linie wird von der Schwarzschild-Zeitkoordinate nicht erreicht t . Es ist ein Problem mit den Koordinaten, nicht mit der Raumzeit. Die wahre zukünftige Singularität ist die Hyperbel für r = 0
Wie wird es nicht erreicht, wenn es von Objekten überquert werden muss, die auf 'radialen zeitähnlichen' und lichtähnlichen Geodäten reisen, um zur Hyperbel zu gelangen? r = 0 ?
@KamilSzot, recherchieren Sie bitte etwas über die Schwarzschild- Koordinaten und insbesondere darüber, wie sie am Horizont singulär sind, obwohl die Raumzeit dort vollkommen regelmäßig ist. Der Kommentarbereich ist nicht für eine ausführliche Diskussion gedacht. Ich empfehle, dass Sie etwas recherchieren und dann, wenn es immer noch nicht klar ist, eine Frage an die Community stellen.
OK. Nur eine Zusatzfrage. Was ist t in t = ? Wie heißt es? Ich verstehe, es ist eine Art Zeit, aber wie heißt es genau? Es hilft mir bei der Suche. Ist es "Schwarzschild-Zeitkoordinate t "? Übrigens frage ich hier nicht nach Raumzeitregelmäßigkeiten oder Singularitäten oder Schwarzen Löchern. Ich frage, wie die Linie, die die Linie kreuzt, beschriftet ist t = könnte davon gesprochen werden, dass diese Linie mit Koordinaten nicht erreicht wird t . Meine Frage bezieht sich nur darauf, was ich mit meinen Augen auf der Zeichnung sehe, ohne Verbindung zur physikalischen Realität, die sie modelliert, oder sogar zum Modell selbst.

Inspiriert von einer ähnlichen Frage habe ich mich zeitgleich mit Rob Jeffries mit diesem Thema beschäftigt. Ärgerlicherweise kam er mir zuvor; aber da ich einen etwas anderen Ansatz verwende und meine Bemühungen nicht umsonst sein sollen, werde ich meine eigene Ableitung posten. Nicht zuletzt dient es als Bestätigung seiner fantastischen Antwort :)

Beginnen wir mit der Angabe der (Region I) Kruskal-Szekeres-Koordinaten

u = f ( r ) cosch ( c t 2 r s ) , v = f ( r ) Sünde ( c t 2 r s ) , f ( r ) = ( r r s 1 ) 1 / 2 e r / 2 r s .

Bekanntlich sind in diesen Koordinaten die Geodäten radial einfallender Lichtstrahlen gerade Linien 45 Winkel. In der Tat, wenn wir einstecken u + v = k in die Gleichungen, mit k eine Konstante, dann von u 2 v 2 = f ( r ) 2 wir finden k ( u v ) = f ( r ) 2 , so dass

k exp ( c t 2 r s ) = f ( r ) = ( r r s 1 ) 1 / 2 exp ( r 2 r s ) ,
oder
c t r s = ln ( k 2 ) r r s ln ( r r s 1 ) ,
die in der Tat die Geodäten eines radial einfallenden Photons sind, mit k = f ( r 0 , γ ) und r 0 , γ die Anfangsposition des Photons bei t = 0 .

Nehmen wir nun an, wir hätten ein radial einfallendes Objekt, das an einer Position in Ruhe beginnt r 0 bei t = 0 . Welche radial einfallenden Photonen erreichen das Objekt, bevor es den Ereignishorizont überquert? Um dies zu beantworten, werden wir versuchen, die Geodäte eines radial einfallenden Photons so abzuleiten, dass es das Objekt genau am Ereignishorizont einholt.

Die Geodäte eines radial einfallenden Objekts kann in der Form geschrieben werden (Misner, Thorne & Wheeler Gl. (31.10), Pag. 824)

r = r 0 2 ( 1 + cos η ) = r 0 cos 2 η / 2 , c τ r s = 1 2 ( r 0 r s ) 3 / 2 ( η + Sünde η ) , c t r s = ln ( r 0 / r s 1 + bräunen η / 2 r 0 / r s 1 bräunen η / 2 ) + ( r 0 r s 1 ) 1 / 2 ( η + r 0 2 r s ( η + Sünde η ) ) .
Es ist auch aufschlussreich, die (dimensionslose) Gesamtenergie des Objekts einzuführen
E = E m c 2 = ( 1 r s r ) d t d τ .
Radialbahnen erfüllen die Gleichung
( d r c d τ ) 2 = E 2 ( 1 r s r ) ,
also wenn das Objekt an der Position ruht r 0 bei t = τ = 0 , dann
E = 1 r s r 0 .
Die Gleichung für t ( η ) kann somit umgeschrieben werden als
c t r s = ln ( E + 1 E 2 bräunen η / 2 E 1 E 2 bräunen η / 2 ) + E 1 E 2 ( η + η + Sünde η 2 ( 1 E 2 ) ) .
Als nächstes werde ich diesem Artikel folgen (der einige Fehler enthält), um abzuleiten, wie sich diese Gleichung verhält r nähert sich dem Ereignishorizont. Wir schreiben
r = r s ( 1 + ε ) , ε 0.
In der Nähe des Ereignishorizonts können wir Terme höherer Ordnung ignorieren ε , so dass
cos 2 η / 2 = ( 1 + ε ) r s r 0 = ( 1 + ε ) ( 1 E 2 ) = 1 E 2 + ε ( 1 E 2 ) , Sünde 2 η / 2 = E 2 ε ( 1 E 2 ) ,
und
( 1 E 2 ) bräunen 2 η / 2 = E 2 ε ( 1 E 2 ) ( 1 + ε ) [ E 2 ε ( 1 E 2 ) ] ( 1 ε ) E 2 ε ( 1 E 2 ) ε ) E 2 = E 2 ε .
Deswegen,
E + 1 E 2 bräunen η / 2 E ( 1 + 1 ε / E 2 ) 2 E ε 2 E 2 E ,
und
E 1 E 2 bräunen η / 2 E ( 1 1 ε / E 2 ) ε 2 E ,
Damit endlich, wie r r s ,
c t s r s ln ( 4 E 2 ε ) + E 1 E 2 ( η s + η s + Sünde η s 2 ( 1 E 2 ) ) ,
mit η s der Wert von η am Ereignishorizont. Seit cosch ( x ) = Sünde ( x ) e x / 2 wie x , werden die Kruskal-Szekeres-Koordinaten des Objekts am Ereignishorizont (seit t )
u s 2 = v s 2 = 1 4 f ( r s ) 2 exp ( c t s r s ) = ε e 4 exp ( c t s r s ) = e E 2 exp [ E 1 E 2 ( η s + η s + Sünde η s 2 ( 1 E 2 ) ) ] ,
oder
u s = v s = e E exp [ E 2 1 E 2 ( η s + η s + Sünde η s 2 ( 1 E 2 ) ) ] = e r s r 0 ( r 0 r s 1 ) 1 / 2 exp [ 1 2 ( r 0 r s 1 ) 1 / 2 ( η s + r 0 2 r s ( η s + Sünde η s ) ) ] .
Die entsprechenden Koordinaten für ein radial einfallendes Photon genügen u s + v s = k b , für einen Grenzwert k b . Deswegen k b = 2 u s und wir finden die entsprechende Null-Geodäte
c t r s = ln ( k b 2 ) r r s ln ( r r s 1 ) .
Wir könnten das lösen r bei t = 0 , was einen Randradius ergibt r b jenseits dessen radial einfallende Photonen das Objekt nicht einholen können, bevor es den Ereignishorizont überquert. Alternativ könnten wir uns anschließen r = r 0 und fragen Sie, was die maximale Zeit Δ t ist so, dass Photonen ausgesendet werden r 0 Vor t = Δ t kann das Objekt noch einholen. Wir finden
c Δ t r s = ln ( k b 2 ) r 0 r s ln ( r 0 r s 1 ) = 1 + ln ( 4 r s r 0 ) + [ ( r 0 r s 1 ) 1 / 2 ( η s + r 0 2 r s ( η s + Sünde η s ) ) ] r 0 r s ,
das ist genau das gleiche Ergebnis wie von Rob Jeffries angegeben.

Ich habe ein Diagramm erstellt, um die Ergebnisse in Schwarzschild- und Kruskal-Szekeres-Koordinaten zu visualisieren:

Geben Sie hier die Bildbeschreibung ein

Die blaue Kurve ist die Geodäte eines Objekts im Ruhezustand t = 0 (hier, r 0 = 2 r s ). Die orange Kurve ist die Geodäte eines Photons, das sich in Position befindet r 0 bei t = 0 . Die rote Kurve ist die Geodäte, die ich in diesem Beitrag abgeleitet habe. Es beginnt bei Position r b bei t = 0 und holt das Objekt direkt am Ereignishorizont ein. Geodäten von Photonen, die zwischen der orangefarbenen und der roten Kurve liegen (ich habe zwei gezeichnet, die gestrichelten Kurven), werden das Objekt einholen, Geodäten jenseits der roten Kurve nicht.

Verwirrt durch das rechte Diagramm. Ist das der Weg eines freifallenden Objekts in KS-Koordinaten?
@RobJeffries Ja, es ist dasselbe Objekt wie das linke Diagramm. Es überquert die EH (at u s = 9.25 ), wenn auch in einem sehr flachen Winkel. Ich bin mir nicht sicher, warum das so ist. Alles andere wird aber überprüft.
Siehe meine wichtige Bearbeitung. Ich habe jetzt (glaube ich) die zusätzliche verfügbare Zeit berücksichtigt, während das Objekt herunterfällt r s zur Singularität. Dies sollte enthalten sein und ist eigentlich das, was wichtig ist, um die Frage zu beantworten. Es ist viel einfacher zu rechnen!

Um die hervorragenden Antworten oben zu ergänzen, finden Sie hier ein Raumzeitdiagramm in Gullstrand-Painleve- oder "Regen" -Koordinaten. Dies ist aus dem großartigen und zugänglichen Buch Exploring Black Holes (2000) von Taylor & Wheeler, § B .6 . Ihre Metapher „Regen“ meint ein Testteilchen mit Masse, das zunächst weit entfernt vom Schwarzen Loch aus der Ruhe gefallen ist. Betrachten Sie sie als Astronauten / Beobachter für dieses Problem.

t Regen ist die Eigenzeit eines Regentropfens, die als Koordinate verwendet wird. r ist die übliche Krümmungskoordinate wie in Schwarzschild[-Droste]-Koordinaten, und M ist die Masse des Schwarzen Lochs. Das Diagramm zeigt, dass die meisten "Lichtimpulse" niemals einen bestimmten "Regenkolben" einholen; insbesondere werden sie das Ende des Universums nicht sehen.

Geben Sie hier die Bildbeschreibung ein

Nein. Das Schwarze Loch wird in endlicher Zeit vollständig verdampfen, sodass es am Ende des Universums nicht mehr existieren wird.

Ich war nicht der Downvoter, aber das ist nicht wirklich eine Antwort. Die Frage wurde im Zusammenhang mit der klassischen Schwerkraft gestellt.
@Ben Crowell, dann ist diese Theorie nicht auf einen solchen Zeitraum anwendbar.

Ihre Frage beruht auf einer gewissen Verwirrung mit dem Raumzeitkonzept eines Schwarzen Lochs. Sie müssen zwischen Ihrem Koordinatensystem und dem, was Sie sehen, unterscheiden. Beides sind unterschiedliche Konzepte: Ein einfaches Beispiel ist ein Minkowski-Raum: Wenn ein Minkowski-Diagramm Ihre Koordinaten darstellt, erhalten Sie eine vierdimensionale Ansicht der gesamten Raumzeit. Was Sie dagegen sehen, sind Elemente, die sich auf Ihrem in die Vergangenheit weisenden Lichtkegel befinden.

In der Nähe eines Schwarzen Lochs müssen wir dieselbe Unterscheidung dieses zweifachen Konzepts anwenden, die im folgenden Kruskal-Diagramm gezeigt werden kann, wobei ein einfallendes Teilchen A und ein Teilchen außerhalb von B verbleiben:

Geben Sie hier die Bildbeschreibung ein

Die Zeitkoordinaten eines weit entfernten Beobachters werden durch die Linien angezeigt, die durch das Zentrum gehen: t = 0, t = 1, t = 2, begrenzt durch den Ereignishorizont mit t = . Gemäß diesen Zeitkoordinaten wird das einfallende Teilchen niemals den Horizont erreichen. Und umgekehrt, wenn sich A dem Horizont nähert, nähert sich die Uhr eines außenstehenden Beobachters dem Ende der Zeit.

Vielleicht ist das der Grund, warum du deine Frage gestellt hast. Aber Ihre Frage lautet nicht: Wie ist die Position in Bezug auf die Koordinaten eines äußeren Beobachters, sondern: Was sieht das einfallende Teilchen, und für diese Frage müssen Sie sich (wie in anderen Antworten gezeigt) auf das kleine 45 ° - beziehen Pfeile zwischen den kommunizierenden Teilchen A und B. Die 3 diagonalen Pfeile von unten nach links zeigen, dass B an einem bestimmten Punkt ist, an dem A den Ereignishorizont berührt.